Maximize Return: Find A to Maximize Contract Award

  • Thread starter Thread starter ptlnguyen
  • Start date Start date
Click For Summary
The discussion centers on maximizing the contract award represented by the function f(x) = 1/2 - 1/4|A - x|, where x is the delivery date. Participants are trying to determine the optimal value of A to maximize this award, particularly within the constraints of 5 < x < 7, which offers a $500 reward. There is confusion regarding how the reward condition affects the function and whether it applies only outside that range. Questions arise about whether the maximization depends on the value of x or if it should consider a probability distribution for x. Clarification is needed on the relationship between A and the specified range for x.
ptlnguyen
Messages
5
Reaction score
0

Homework Statement



Given:
f(x) = 1/2 - 1/4|A - x| where f(x) is total contract award, x = deliver date

if 5 < x < 7 then get a reward $500


Homework Equations



Find A such that it maximizes the contract award

The Attempt at a Solution


My approach: take derivative of f(x) and set it = 0 and solve for A
Am I correct? Thanks
 
Physics news on Phys.org
ptlnguyen said:

Homework Statement



Given:
f(x) = 1/2 - 1/4|A - x| where f(x) is total contract award, x = deliver date

if 5 < x < 7 then get a reward $500


Homework Equations



Find A such that it maximizes the contract award

The Attempt at a Solution


My approach: take derivative of f(x) and set it = 0 and solve for A
Am I correct? Thanks

What did you get?

Also, I'm moving this to the Calculus & Beyond section.
 
ptlnguyen said:
if 5 < x < 7 then get a reward $500
I don't understand how that item of information fits in. Does it mean that the given formula for f(x) only applies outside that range?
Find A such that it maximizes the contract award
Won't that depend on x? Or is it supposed to maximise the average award given some prob dist for x? (There's a very similar problem in another recent thread, and it's just as unclear.)
 
Question: A clock's minute hand has length 4 and its hour hand has length 3. What is the distance between the tips at the moment when it is increasing most rapidly?(Putnam Exam Question) Answer: Making assumption that both the hands moves at constant angular velocities, the answer is ## \sqrt{7} .## But don't you think this assumption is somewhat doubtful and wrong?

Similar threads

Replies
1
Views
2K
  • · Replies 8 ·
Replies
8
Views
2K
  • · Replies 12 ·
Replies
12
Views
2K
  • · Replies 2 ·
Replies
2
Views
2K
  • · Replies 12 ·
Replies
12
Views
2K
  • · Replies 7 ·
Replies
7
Views
2K
Replies
11
Views
2K
Replies
10
Views
1K
  • · Replies 7 ·
Replies
7
Views
2K
  • · Replies 27 ·
Replies
27
Views
3K